Last visit was: 01 May 2024, 22:23 It is currently 01 May 2024, 22:23

Close
GMAT Club Daily Prep
Thank you for using the timer - this advanced tool can estimate your performance and suggest more practice questions. We have subscribed you to Daily Prep Questions via email.

Customized
for You

we will pick new questions that match your level based on your Timer History

Track
Your Progress

every week, we’ll send you an estimated GMAT score based on your performance

Practice
Pays

we will pick new questions that match your level based on your Timer History
Not interested in getting valuable practice questions and articles delivered to your email? No problem, unsubscribe here.
Close
Request Expert Reply
Confirm Cancel
SORT BY:
Date
Magoosh GMAT Instructor
Joined: 28 Dec 2011
Posts: 4452
Own Kudos [?]: 28589 [22]
Given Kudos: 130
Most Helpful Reply
User avatar
Manager
Manager
Joined: 19 Apr 2013
Posts: 55
Own Kudos [?]: 186 [14]
Given Kudos: 9
Concentration: Entrepreneurship, Finance
GMAT Date: 06-05-2015
GPA: 3.88
WE:Programming (Computer Software)
Send PM
avatar
Intern
Intern
Joined: 30 Apr 2010
Posts: 14
Own Kudos [?]: 76 [9]
Given Kudos: 2
Send PM
General Discussion
avatar
Intern
Intern
Joined: 19 Oct 2013
Posts: 8
Own Kudos [?]: 56 [0]
Given Kudos: 13
Location: United States
Concentration: Finance, Technology
GMAT Date: 11-06-2013
GPA: 3.5
WE:Engineering (Investment Banking)
Send PM
Re: Is |x - 6| > 2? (1) |x - 4| > 3 (2) |x - 8| > 1 [#permalink]
Spunkerspawn wrote:
Is |x - 6| > 2? What this question is basically asking us is if x > 8 or if x < 4 since any value between 4 and 8 would make the inequality not hold (try it: x = 5 |5-6| = |-1| = 1 < 2)

Statement #1: |x - 4| > 3 tells us that x > 7 or x < 1, not enough info to say whether x > 8 (could be 7.2 for example) not sufficient.
Statement #2: |x - 8| > 1 tells us that x > 9 or x < 7, not enough info to say whether x < 4, not sufficient

Taken together (1) & (2) x < 1 and x > 9: any value of x will make the inequality true so yes |x - 6| > 2. Sufficient

Answer: C

This has made it super clear!! Thanks a lot!!
User avatar
Manager
Manager
Joined: 22 Feb 2009
Posts: 109
Own Kudos [?]: 527 [0]
Given Kudos: 148
Send PM
Re: Is |x - 6| > 2? (1) |x - 4| > 3 (2) |x - 8| > 1 [#permalink]
bhatiamanu05 wrote:
Is |x - 6| > 2?
Statement #1: |x - 4| > 3
Statement #2: |x - 8| > 1

the question statement : X-6>2 and X-6<-2 ==> X>8 and X<4

Statement #1 : X-4>3 and X-4<-3 ===> x>7 and X<1
Statement #2: X-8>1 and X-8<-1 ==> X>9 and X<7

So combining 1 and 2 we get answer as X>9 and X<1 for this the question statement holds true.

So C is the answer.

+1Kudos if you like



Thanks
AB


I think your solution is clear enough.
I use different way but the result is the same
|x-4| = x - 4 when x>= 4 or |x-4| = 4 - x when x < 4, --> I have x>7 and x<1

Already +1 Kudos for you
User avatar
Manager
Manager
Joined: 06 Mar 2014
Posts: 166
Own Kudos [?]: 475 [0]
Given Kudos: 84
Location: India
GMAT Date: 04-30-2015
Send PM
Re: Is |x - 6| > 2? (1) |x - 4| > 3 (2) |x - 8| > 1 [#permalink]
bhatiamanu05 wrote:
Is |x - 6| > 2?
Statement #1: |x - 4| > 3
Statement #2: |x - 8| > 1

the question statement : X-6>2 and X-6<-2 ==> X>8 and X<4

Statement #1 : X-4>3 and X-4<-3 ===> x>7 and X<1
Statement #2: X-8>1 and X-8<-1 ==> X>9 and X<7

So combining 1 and 2 we get answer as X>9 and X<1 for this the question statement holds true.

So C is the answer.

+1Kudos if you like

Thanks
AB


SO IF X =3 it follows x<4 but is not supported by our solution x<1.
??
Director
Director
Joined: 12 Nov 2016
Posts: 569
Own Kudos [?]: 118 [0]
Given Kudos: 167
Location: United States
Schools: Yale '18
GMAT 1: 650 Q43 V37
GRE 1: Q157 V158
GPA: 2.66
Send PM
Re: Is |x - 6| > 2? (1) |x - 4| > 3 (2) |x - 8| > 1 [#permalink]
mikemcgarry wrote:
Is |x - 6| > 2?
Statement #1: |x - 4| > 3
Statement #2: |x - 8| > 1


For a discussion of inequalities and DS questions, as well as an efficient solution for this particular problem, see:
https://magoosh.com/gmat/2013/gmat-quant ... qualities/


Original Stmnt

x-6> 2
X > 8

-lx-6l>2
-x +6 >2
-x > -4
x< 4 - remember to flip

X>8 or x<4

Stmnt 1

x - 4 > 3
x >7

-|x - 4| > 3
-x +4 >3
-x > -1
x <1

Insuff
Stmnt 2

x - 8 > 1
x> 9

-|x - 8| > 1
-x +8 > 1
-x >-7
x< 7

Insuff

Stmnt 1 & 2

x>9 and x>1 - either satisfies the condition

C
Intern
Intern
Joined: 14 Dec 2016
Posts: 31
Own Kudos [?]: 6 [0]
Given Kudos: 150
Location: India
GMAT 1: 200 Q49 V37
GPA: 4
Send PM
Re: Is |x - 6| > 2? (1) |x - 4| > 3 (2) |x - 8| > 1 [#permalink]
Is |x - 6| > 2?
Statement #1: |x - 4| > 3
Statement #2: |x - 8| > 1



So the question basically asks us is if:
x > 8 OR x < 4

Statement #1: |x - 4| > 3 tells us that x > 7 or x < 1--So this statement does satisfy the second condition (x<4)
Statement #2: |x - 8| > 1 tells us that x > 9 or x < 7--So this statement does satisfy the first condition (x>8)

Bunuel Please explain me what I am doing wrong.
Senior PS Moderator
Joined: 26 Feb 2016
Posts: 2873
Own Kudos [?]: 5208 [0]
Given Kudos: 47
Location: India
GPA: 3.12
Send PM
Re: Is |x - 6| > 2? (1) |x - 4| > 3 (2) |x - 8| > 1 [#permalink]
ShashwatPrakash wrote:
Is |x - 6| > 2?
Statement #1: |x - 4| > 3
Statement #2: |x - 8| > 1



So the question basically asks us is if:
x > 8 OR x < 4

Statement #1: |x - 4| > 3 tells us that x > 7 or x < 1--So this statement does satisfy the second condition (x<4)
Statement #2: |x - 8| > 1 tells us that x > 9 or x < 7--So this statement does satisfy the first condition (x>8)

Bunuel Please explain me what I am doing wrong.



Hey ShashwatPrakash

The highlighted portion is your only mistake.
Now, because the range of the inequality is x>8 and x<4, the only possibility is when we
combine the information from both the sentences. Hence, the answer is Option C

Hope this helps you
Manager
Manager
Joined: 04 Feb 2016
Posts: 51
Own Kudos [?]: 24 [0]
Given Kudos: 4
Location: India
Concentration: Technology, Marketing
GMAT 1: 650 Q48 V32
GPA: 4
WE:Sales (Computer Software)
Send PM
Re: Is |x - 6| > 2? (1) |x - 4| > 3 (2) |x - 8| > 1 [#permalink]
x<4 | x >8 => Yes.

1. x < 1 | x > 7 => Yes | May be => insufficient.
2. x < 7 | x > 9 => May be | Yes.
1+2 => common regions x < 1 | x > 9 => Yes | Yes.

I used the number line approach.
VP
VP
Joined: 11 Aug 2020
Posts: 1259
Own Kudos [?]: 202 [0]
Given Kudos: 332
Send PM
Re: Is |x - 6| > 2? (1) |x - 4| > 3 (2) |x - 8| > 1 [#permalink]
bhatiamanu05 wrote:
Is |x - 6| > 2?
Statement #1: |x - 4| > 3
Statement #2: |x - 8| > 1

the question statement : X-6>2 and X-6<-2 ==> X>8 and X<4

Statement #1 : X-4>3 and X-4<-3 ===> x>7 and X<1
Statement #2: X-8>1 and X-8<-1 ==> X>9 and X<7

So combining 1 and 2 we get answer as X>9 and X<1 for this the question statement holds true.

So C is the answer.

+1Kudos if you like

Thanks
AB


mikemcgarry Hi Mike, I did all of the dirty work above and got the exact same solution, but I don't understand how I got there. Can you explain why the statements individually are insufficient? Secondly, can you explain why the final answer is x > 9 and x < 1? Why those particular ranges?
User avatar
Non-Human User
Joined: 09 Sep 2013
Posts: 32772
Own Kudos [?]: 824 [0]
Given Kudos: 0
Send PM
Re: Is |x - 6| > 2? (1) |x - 4| > 3 (2) |x - 8| > 1 [#permalink]
Hello from the GMAT Club BumpBot!

Thanks to another GMAT Club member, I have just discovered this valuable topic, yet it had no discussion for over a year. I am now bumping it up - doing my job. I think you may find it valuable (esp those replies with Kudos).

Want to see all other topics I dig out? Follow me (click follow button on profile). You will receive a summary of all topics I bump in your profile area as well as via email.
GMAT Club Bot
Re: Is |x - 6| > 2? (1) |x - 4| > 3 (2) |x - 8| > 1 [#permalink]
Moderator:
Math Expert
92987 posts

Powered by phpBB © phpBB Group | Emoji artwork provided by EmojiOne